K
Khách

Hãy nhập câu hỏi của bạn vào đây, nếu là tài khoản VIP, bạn sẽ được ưu tiên trả lời.

6 tháng 9 2020

Bài này không đúng nhé. Với a = b = c = 1 thì bất đẳng thức sai. Tuy nhiên bài này đúng theo chiều ngược lại.

7 tháng 9 2020

Ta sẽ chứng minh bất đẳng thức phụ sau đây \(x^2+y^2+z^2\ge xy+yz+zx\)

\(< =>2\left(x^2+y^2+z^2\right)\ge2\left(xy+yz+zx\right)\)

\(< =>2x^2+2y^2+2z^2-2xy-2yz-2zx\ge0\)

\(< =>\left(x-y\right)^2+\left(y-z\right)^2+\left(z-x\right)^2\ge0\)*đúng*

Đặt \(\left\{2a+2b-c;2b+2c-a;2c+2a-b\right\}\rightarrow\left\{x;y;z\right\}\)

Vì a,b,c là ba cạnh của 1 tam giác nên x,y,z dương 

Ta có : \(x^2+y^2+z^2=9\left(a^2+b^2+c^2\right)\)

\(x+y=c+a+4b\)\(y+z=a+b+4c\)\(z+x=b+c+4a\)

Bất đẳng thức cần chứng minh quy về : \(\frac{x^3}{y+z}+\frac{y^3}{x+z}+\frac{z^3}{x+y}\ge\frac{x^2+y^2+z^2}{2}\)

Áp dụng bất đẳng thức AM-GM ta có : 

\(\frac{x^3}{y+z}+\frac{x\left(y+z\right)}{4}\ge2\sqrt{\frac{x^3.x\left(y+z\right)}{\left(y+z\right)4}}=2\sqrt{\frac{x^4}{4}}=2\frac{x^2}{2}=x^2\)

\(\frac{y^3}{x+z}+\frac{y\left(x+z\right)}{4}\ge2\sqrt{\frac{y^3.y\left(x+z\right)}{\left(x+z\right)4}}=2\sqrt{\frac{y^4}{4}}=2\frac{y^2}{2}=y^2\)

\(\frac{z^3}{x+y}+\frac{z\left(x+y\right)}{4}\ge2\sqrt{\frac{z^3.z\left(x+y\right)}{\left(x+y\right)4}}=2\sqrt{\frac{z^4}{4}}=2\frac{z^2}{2}=z^2\)

Cộng theo vế các bất đẳng thức cùng chiều ta được :

\(\frac{x^3}{y+z}+\frac{y^3}{x+z}+\frac{z^3}{x+y}+\frac{x\left(y+z\right)}{4}+\frac{y\left(x+z\right)}{4}+\frac{z\left(x+y\right)}{4}\ge x^2+y^2+z^2\)

\(< =>\frac{x^3}{y+z}+\frac{y^3}{x+z}+\frac{z^3}{x+y}+\frac{xy+yz+zx+xy+yz+zx}{4}\ge x^2+y^2+z^2\)

\(< =>\frac{x^3}{y+z}+\frac{y^3}{x+z}+\frac{z^3}{x+y}+\frac{xy+yz+zx}{2}\ge x^2+y^2+z^2\)

\(< =>\frac{x^3}{y+z}+\frac{y^3}{x+z}+\frac{z^3}{x+y}\ge x^2+y^2+z^2-\frac{xy+yz+zx}{2}\)

Sử dụng bất đẳng thức phụ \(x^2+y^2+z^2\ge xy+yz+zx\)khi đó ta được :

\(\frac{x^3}{y+z}+\frac{y^3}{x+z}+\frac{z^3}{y+x}\ge x^2+y^2+z^2-\frac{x^2+y^2+z^2}{2}\)

\(< =>\frac{x^3}{y+z}+\frac{y^3}{z+x}+\frac{z^3}{x+y}\ge\frac{x^2+y^2+z^2}{2}\left(đpcm\right)\)

Đẳng thức xảy ra khi và chỉ khi \(x=y=z< =>a=b=c\)

Vậy ta có điều phải chứng minh

2 tháng 9 2020

1 bài BĐT rất hay !!!!!!

BẠN PHÁ TOANG RA HẾT NHÁ SAU ĐÓ THÌ ĐƯỢC CÁI NÀY :33333

\(S=15\left(a^3+b^3+c^3\right)+6\left(a^2b+ab^2+b^2c+bc^2+a^2c+ac^2\right)-72abc\)

\(S=9\left(a^3+b^3+c^3\right)+6\left(a^3+b^3+c^3+a^2b+ab^2+b^2c+bc^2+c^2a+ca^2\right)-72abc\)

\(S=9\left(a^3+b^3+c^3\right)+6\left(a+b+c\right)\left(a^2+b^2+c^2\right)-72abc\)

TA ÁP DỤNG BĐT CAUCHY 3 SỐ SẼ ĐƯỢC:

\(\hept{\begin{cases}a+b+c\ge3\sqrt[3]{abc}\\a^2+b^2+c^2\ge3\sqrt[3]{a^2b^2c^2}\end{cases}}\)

=>    \(\left(a+b+c\right)\left(a^2+b^2+c^2\right)\ge9abc\)

=>    \(72abc\le8\left(a+b+c\right)\left(a^2+b^2+c^2\right)\)

=>   \(-72abc\ge-8\left(a+b+c\right)\left(a^2+b^2+c^2\right)\)

=>   \(S\ge9\left(a^3+b^3+c^3\right)+6\left(a+b+c\right)\left(a^2+b^2+c^2\right)-8\left(a+b+c\right)\left(a^2+b^2+c^2\right)\)

=>   \(S\ge9\left(a^3+b^3+c^3\right)-2\left(a+b+c\right)\left(a^2+b^2+c^2\right)\)

=>   \(S\ge9\left(a^3+b^3+c^3\right)-\frac{2}{9}\left(a+b+c\right)\)

TA LẠI TIẾP TỤC ÁP DỤNG BĐT SAU:   \(\left(a+b+c\right)^2\le3\left(a^2+b^2+c^2\right)\Rightarrow\left(a+b+c\right)^2\le\frac{1}{3}\Rightarrow a+b+c\le\sqrt{\frac{1}{3}}\)

=>   \(S\ge9\left(a^3+b^3+c^3\right)-\frac{2}{9}.\sqrt{\frac{1}{3}}\)

TA LẦN LƯỢT ÁP DỤNG BĐT CAUCHY 3 SỐ SẼ ĐƯỢC:

\(a^3+a^3+\left(\sqrt{\frac{1}{27}}\right)^3\ge3a^2.\sqrt{\frac{1}{27}}\)

ÁP DỤNG TƯƠNG TỰ VỚI 2 BIẾN b; c ta sẽ được 1 BĐT như sau: 

=>   \(2\left(a^3+b^3+c^3\right)+3\left(\sqrt{\frac{1}{27}}\right)^3\ge\frac{3}{\sqrt{27}}\left(a^2+b^2+c^2\right)=\frac{3}{\sqrt{27}}.\left(\frac{1}{9}\right)=\frac{\sqrt{3}}{27}\)

=>   \(a^3+b^3+c^3\ge\frac{\left(\frac{\sqrt{3}}{27}-3\left(\sqrt{\frac{1}{27}}\right)^3\right)}{2}\)

=>   \(S\ge\frac{9\left(\frac{\sqrt{3}}{27}-3\left(\sqrt{\frac{1}{27}}\right)^3\right)}{2}-\frac{2}{9}.\sqrt{\frac{1}{3}}\)

=>   \(S\ge\frac{1}{\sqrt{3}}\)

VẬY TA CÓ ĐPCM.

DẤU "=" XẢY RA <=>   \(a=b=c=\sqrt{\frac{1}{27}}\)

15 tháng 5 2018

Mình nhầm, phải là \(\le\frac{1}{3}\)mọi người làm giúp mình với mình cần gấp

1 tháng 8 2020

Theo BĐT Cauchy Schwarz và các biến đổi cơ bản ta dễ có được:
\(\frac{a^2}{\left(2a+b\right)\left(2a+c\right)}=\frac{a^2}{2a\left(a+b+c\right)+2a^2+bc}=\frac{1}{9}\left[\frac{\left(2a+a\right)^2}{2a\left(a+b+c\right)+2a^2+bc}\right]\)

\(\le\frac{1}{9}\left[\frac{4a^2}{2a\left(a+b+c\right)}+\frac{a^2}{2a^2+bc}\right]=\frac{1}{9}\left(\frac{2a}{a+b+c}+\frac{a^2}{2a^2+bc}\right)\)

\(\Rightarrow LHS\le\frac{1}{9}\left(2+\frac{a^2}{2a^2+bc}+\frac{b^2}{2b^2+ca}+\frac{c^2}{2c^2+ab}\right)\)

Tiếp tục theo BĐT Cauchy Schwarz dạng Engel:

\(\frac{a^2}{a^2+2bc}+\frac{b^2}{b^2+2ca}+\frac{c^2}{c^2+2ab}\ge\frac{\left(a+b+c\right)^2}{\left(a+b+c\right)^2}=1\)

Ta thực hiện phép đổi biến thì:

\(\frac{ab}{ab+2c^2}+\frac{bc}{bc+2a^2}+\frac{ca}{ca+2b^2}\ge1\)

Đến đây là phần của bạn

9 tháng 10 2017

Ta có \(a^2b^2+b^2c^2+c^2a^2\geq a^2b^2c^2\Leftrightarrow \frac{1}{a^2}+\frac{1}{b^2}+\frac{1}{c^2}\geq 1\)

BĐT cần chứng minh tương đương với \(\frac{\frac{1}{c^3}}{\frac{1}{a^2}+\frac{1}{b^2}}+\frac{\frac{1}{b^3}}{\frac{1}{a^2}+\frac{1}{c^2}}+\frac{\frac{1}{a^3}}{\frac{1}{b^2}+\frac{1}{c^2}}\geq \frac{\sqrt{3}}{2}\)

Đặt \((\frac{1}{a},\frac{1}{b},\frac{1}{c})=(x,y,z)\). Bài toán trở thành: 

Cho \(x,y,z>0|x^2+y^2+z^2\geq 1\). CMR \(P=\frac{x^3}{y^2+z^2}+\frac{y^3}{z^2+x^2}+\frac{z^3}{x^2+y^2}\geq \frac{\sqrt{3}}{2}\)

Lời giải:

 Áp dụng BĐT Cauchy -Schwarz:

\(P=\frac{x^4}{xy^2+xz^2}+\frac{y^4}{yz^2+yx^2}+\frac{z^4}{zx^2+zy^2}\geq \frac{(x^2+y^2+^2)^2}{x^2(y+z)+y^2(x+z)+z^2(x+y)}\) (1)

Không mất tính tổng quát, giả sử \(x\geq y\geq z\Rightarrow x^2\geq y^2\geq z^2\) 

Và \(y+z\leq z+x\leq x+y\). Khi đó, áp dụng BĐT Chebyshev: 

\(3[x^2(y+z)+y^2(x+z)+z^2(x+y)]\leq (x^2+y^2+z^2)(y+z+x+z+x+y)\)

\(\Leftrightarrow x^2(y+z)+y^2(x+z)+z^2(x+y)\leq \frac{2(x^2+y^2+z^2)(x+y+z)}{3}\)

Theo hệ quả của BĐT Am-Gm thì: \((x+y+z)^2\leq 3(x^2+y^2+z^2)\Rightarrow x+y+z\leq \sqrt{3(x^2+y^2+z^2)}\)

\(\Rightarrow x^2(y+z)+y^2(x+z)+z^2(x+y)\leq \frac{2(x^2+y^2+z^2)\sqrt{3(x^2+y^2+z^2)}}{3}\) (2)

Từ (1),(2) suy ra \(P\geq \frac{3(x^2+y^2+z^2)^2}{2(x^2+y^2+z^2)\sqrt{3(x^2+y^2+z^2)}}=\frac{\sqrt{3(x^2+y^2+z^2)}}{2}\geq \frac{\sqrt{3}}{2}\)

Ta có đpcm

Dáu bằng xảy ra khi \(x=y=z=\frac{1}{\sqrt{3}}\Leftrightarrow a=b=c=\sqrt{3}\)

5 tháng 5 2020

Đặt \(x=\frac{1}{a};y=\frac{1}{b};z=\frac{1}{c}\)

Khi đó giả thiết được viết lại là \(x^2+y^2+z^2\ge1\)và ta cần chứng minh \(\frac{x^3}{y^2+z^2}+\frac{y^3}{z^2+x^2}+\frac{z^3}{x^2+y^2}\ge\frac{\sqrt{3}}{2}\)(*)

Áp dụng BĐT Bunhiacopxki dạng phân thức, ta được:

\(VT_{\left(^∗\right)}=\frac{x^4}{x\left(y^2+z^2\right)}+\frac{y^4}{y\left(z^2+x^2\right)}+\frac{z^4}{z\left(x^2+y^2\right)}\)\(\ge\frac{\left(x^2+y^2+z^2\right)^2}{x\left(y^2+z^2\right)+y\left(z^2+x^2\right)+z\left(x^2+y^2\right)}\)

Đến đây ta đi chứng minh \(\frac{\left(x^2+y^2+z^2\right)^2}{x\left(y^2+z^2\right)+y\left(z^2+x^2\right)+z\left(x^2+y^2\right)}\ge\frac{\sqrt{3}}{2}\)

\(\Leftrightarrow2\left(x^2+y^2+z^2\right)^2\)\(\ge\sqrt{3}\left[x\left(y^2+z^2\right)+y\left(z^2+x^2\right)+z\left(x^2+y^2\right)\right]\)

Ta có: \(x\left(y^2+z^2\right)=\frac{1}{\sqrt{2}}\sqrt{2x^2\left(y^2+z^2\right)\left(y^2+z^2\right)}\)\(\le\frac{1}{\sqrt{2}}\sqrt{\left(\frac{2x^2+y^2+z^2+y^2+z^2}{3}\right)^3}\)

\(=\frac{2\sqrt{3}}{9}\left(x^2+y^2+z^2\right)\sqrt{x^2+y^2+z^2}\)

Tương tự ta có: \(y\left(z^2+x^2\right)\le\frac{2\sqrt{3}}{9}\left(x^2+y^2+z^2\right)\sqrt{x^2+y^2+z^2}\)

\(z\left(x^2+y^2\right)\le\frac{2\sqrt{3}}{9}\left(x^2+y^2+z^2\right)\sqrt{x^2+y^2+z^2}\)

Cộng theo vế của 3 BĐT trên, ta được: 

\(\text{∑}_{cyc}\left[x\left(y^2+z^2\right)\right]\le\frac{2\sqrt{3}}{3}\left(x^2+y^2+z^2\right)\sqrt{x^2+y^2+z^2}\)

\(\Leftrightarrow\sqrt{3}\text{∑}_{cyc}\left[x\left(y^2+z^2\right)\right]\le2\left(x^2+y^2+z^2\right)\sqrt{x^2+y^2+z^2}\)

Cuối cùng ta cần chứng minh được

\(2\left(x^2+y^2+z^2\right)\sqrt{x^2+y^2+z^2}\le2\left(x^2+y^2+z^2\right)^2\)

\(\Leftrightarrow x^2+y^2+z^2\ge1\)(đúng)

Đẳng thức xảy ra khi \(x=y=z=\frac{1}{\sqrt{3}}\Rightarrow a=b=c=\sqrt{3}\)

AH
Akai Haruma
Giáo viên
30 tháng 5 2020

Lời giải:

Áp dụng BĐT Bunhiacopkxy:

\((2a^2+b^2)(2a^2+c^2)=(a^2+a^2+b^2)(a^2+c^2+a^2)\geq (a^2+ac+ab)^2\)

\(=[a(a+b+c)]^2\)

\(\Rightarrow \frac{a^3}{(2a^2+b^2)(2a^2+c^2)}\leq \frac{a^3}{[a(a+b+c)]^2}=\frac{a}{(a+b+c)^2}\)

Hoàn toàn tương tự với các phân thức còn lại và cộng theo vế thu được:

\(\sum \frac{a^3}{(2a^2+b^2)(2a^2+c^2)}\leq \frac{a+b+c}{(a+b+c)^2}=\frac{1}{a+b+c}\) (đpcm)

Dấu "=" xảy ra khi $a=b=c$

28 tháng 9 2017

moi nguoi oi hom truoc minh hoc tap hop cac so TN do thi co cua minh day nhu sau 

vd: A={xeN/3<x<9}

thi minh liet ke ra la A=4,5,6,7,8 nhung sua bai lai ko dung 

co sua nhu vay A=3,4,5,6,7,8

ko biet hay sai mong ae giup minh

30 tháng 9 2017

Áp dụng BĐT Cô-si \(ab\le\frac{\left(a+b\right)}{4}^2\)

=> \(\left(2a+b\right)\left(2c+b\right)\le\frac{4\left(a+b+c\right)^2}{4}=\left(a+b+c\right)^2\)

=> \(\frac{1}{\left(2a+b\right)\left(2c+b\right)}\ge\frac{1}{\left(a+b+c\right)^2}\)

Mấy cái kia làm tương tự cậu nhé 

Dấu "=" xảy ra khi và chỉ khi a=b=c=1

1 tháng 3 2020

\(\frac{a+b+c}{9}\)nha

Đặt \(P=\frac{a^3}{\left(b+2c\right)^2}+\frac{b^3}{\left(c+2a\right)^2}+\frac{c^3}{\left(a+2b\right)^2}\)

Áp dụng bđt AM-GM cho 3 số dương a,b,c ta được:
\(\frac{a^3}{\left(b+2c\right)^2}+\frac{b+2c}{27}+\frac{b+2c}{27}\ge3\sqrt[3]{\frac{a^3}{\left(b+2c\right)^2}.\frac{b+2c}{27}.\frac{b+2c}{27}}=\frac{a}{3}\)

\(\frac{b^3}{\left(c+2a\right)^2}+\frac{c+2a}{27}+\frac{c+2a}{27}\ge3\sqrt[3]{\frac{b^3}{\left(c+2a\right)^2}.\frac{c+2a}{27}.\frac{c+2a}{27}}=\frac{b}{3}\)

\(\frac{c^3}{\left(a+2b\right)^2}+\frac{a+2b}{27}+\frac{a+2b}{27}\ge3\sqrt[3]{\frac{c^3}{\left(a+2b\right)^2}.\frac{a+2b}{27}.\frac{a+2b}{27}}=\frac{c}{3}\)

Cộng từng vế ta được: 

\(P+\)\(\frac{6\left(a+b+c\right)}{27}\ge\frac{a+b+c}{3}\)

\(\Rightarrow P\ge\frac{a+b+c}{9}\)

Dấu"="xảy ra \(\Leftrightarrow a=b=c\)

AH
Akai Haruma
Giáo viên
3 tháng 3 2017

Lời giải:

Để ý rằng:

\(\frac{4a^2+(b-c)^2}{2a^2+b^2+c^2}=\frac{2(2a^2+b^2+c^2)-2(b^2+c^2)+(b-c)^2}{2a^2+b^2+c^2}=2-\frac{(b+c)^2}{2a^2+b^2+c^2}\)

Biến đổi tương tự với các phân thức còn lại:

\(\Rightarrow \text{VT}=6-\underbrace{\left[\frac{(b+c)^2}{2a^2+b^2+c^2}+\frac{(c+a)^2}{2b^2+a^2+c^2}+\frac{(a+b)^2}{2c^2+a^2+b^2}\right]}_{N}\)

Ta muốn CM \(\text{VT}\geq 3\Leftrightarrow N\leq 3\) . Thật vậy:

Áp dụng BĐT Cauchy-Schwarz:

\(\frac{(b+c)^2}{2a^2+b^2+c^2}\leq \frac{b^2}{a^2+b^2}+\frac{c^2}{a^2+c^2}\). Tương tự như vậy:

\(\left\{\begin{matrix} \frac{(a+c)^2}{2b^2+a^2+c^2}\leq \frac{a^2}{a^2+b^2}+\frac{c^2}{c^2+b^2}\\ \frac{(a+b)^2}{2c^2+a^2+b^2}\leq \frac{a^2}{a^2+c^2}+\frac{b^2}{b^2+c^2}\end{matrix}\right.\)

Cộng theo vế thu được \(N\leq \frac{a^2+b^2}{a^2+b^2}+\frac{b^2+c^2}{b^2+c^2}+\frac{c^2+a^2}{c^2+a^2}=3\)

CM hoàn tất. Dấu bằng xảy ra khi \(a=b=c>0\)

8 tháng 3 2017

#Akai...: Cho em hỏi, đoạn đầu chị ghi "để ý rằng" khi trình bày ra thì mik ghi như thế nào ạ. Không lẽ lại ghi "để ý rằng"